Please confirm topic selection

Are you sure you want to trigger topic in your Anconeus AI algorithm?

Please confirm action

You are done for today with this topic.

Would you like to start learning session with this topic items scheduled for future?

Review Question - QID 214554

In scope icon M 1 A
QID 214554 (Type "214554" in App Search)
A 53-year-old man presents to the emergency department with lightheadedness and palpitations. He has had similar episodes in the past, but they have never lasted this long. His heart has been "racing" for 2 hours. He denies loss of consciousness, chest pain, weakness, or any changes in speech. His temperature is 97.4°F (36.3°C), blood pressure is 135/68 mmHg, pulse is 131/min, and respirations are 19/min. A stat EKG is performed and shown in Figure A. Which of the following is the most likely rate limiting portion of the cardiac conduction system?
  • A
  • A